PT#53, S#3 (LR), Q#25 (problem here?) Forum

Prepare for the LSAT or discuss it with others in this forum.
Post Reply
sighsigh

Bronze
Posts: 263
Joined: Wed Oct 20, 2010 8:47 pm

PT#53, S#3 (LR), Q#25 (problem here?)

Post by sighsigh » Fri Mar 15, 2013 3:17 am

(1) No issue addressed at meeting relevant to person => person not required to attend meeting.
---
(2) Terry not required to attend meeting.

Sufficient assumption: No issue addressed at meeting relevant to Terry. We'll state this as: (3) issue addressed at meeting => issue ~relevant to Terry.

That part is easy. But here's the part I don't get. (C), the correct answer, states: issue relevant to Terry => issue ~relevant to a majority of those attending meeting.

How do you get from (3) to (C)? Well, they expect you to do so via the first sentence of the stimulus: Meetings address only those issues relevant to a majority of those attending. I.e. (4) Issue addressed at meeting => issue relevant to a majority of those attending meeting.

But I can't see any way (3) and (4) can be combined to infer (C)! Someone please tell me what I'm doing wrong.

Legallybronzed180

New
Posts: 32
Joined: Tue Jan 29, 2013 9:50 pm

Re: PT#53, S#3 (LR), Q#25 (problem here?)

Post by Legallybronzed180 » Fri Mar 15, 2013 7:33 am

You are missing the first sentence of the passage - only those issues relevant to a majority are addressed
If addressed then relevant to the majority
If not relevant to the majority then not addressed
Combine that with if no issues are relevant then not required to attend

You get to answer c if relevant to Terry then not relevant to majority and if issues not relevant to majority then not addressed
Therefore we know no issue relevant to Terry is discussed at the meeting and it is is sufficient she isn't required to attend

If you need negate c there are issues relevant to the majority and to Terry then it wouldn't be sufficient that she isn't required to attend the meeting

Hope it helps

sighsigh

Bronze
Posts: 263
Joined: Wed Oct 20, 2010 8:47 pm

Re: PT#53, S#3 (LR), Q#25 (problem here?)

Post by sighsigh » Fri Mar 15, 2013 10:45 pm

Thanks for responding, but I don't understand your explanation. I didn't miss the first sentence. Could you explain more clearly pls?

User avatar
Power Clean

New
Posts: 64
Joined: Sat Dec 01, 2012 12:35 am

Re: PT#53, S#3 (LR), Q#25 (problem here?)

Post by Power Clean » Wed Mar 27, 2013 12:49 pm

I'll try a crack at it: you're not trying to get to C, you're trying to justify the conclusion (the assumption) using one of the answer choices (C).

Issue addressed --> Relevant(majority)

~Relevant(person)* --> ~Attend(person) *NO ISSUE

Right away we can start looking for something to bridge not relevant to a person and relevant to the majority. This way, anything that is addressed at the meeting would produce, necessarily, Terry not having to attend. Something to put her at odds with the majority then.

Answer A: Just because her presentation would be irrelevant to the majority does not mean that NONE of the issues addressed throughout the meeting will be relevant to her. It might mean her issue isn't presented, but we don't know whether or not she'll attend based on this.

Answer B: Who cares?

Answer C: Relevant(terry) --> ~Relevant(majority). This is the bridge we're looking for to justify the conclusion. The contrapositive would be Relevant(majority)--> ~Relevant(terry), which means: If an issue is addressed --> relevant to the majority --> not relevant to terry --> she doesn't have to attend.

Answer D: So she changes the balance of what is relevant to the majority, who cares. It's still relevant to the majority, but who knows if it's relevant to Terry, which will determine whether or not she attends.

Answer E: The majority of issues might not be, but that doesn't preclude the possibility of at least 1 being relevant, and forcing her to attend.

Want to continue reading?

Register now to search topics and post comments!

Absolutely FREE!


Post Reply

Return to “LSAT Prep and Discussion Forum”